अंतर्जात वृद्धि: सीआरआरए उपयोगिता के साथ संतुलित विकास पथ


5

मुझे स्पिलओवर के कारण अंतर्जात वृद्धि का एक मॉडल मिला है।

$ \ Textbf {मॉडल:} $ $$ K_t = \ frac {1} {n} \ sum_ {t = 1} ^ nk_t $$ इस मॉडल में, $ k_t $ को एजेंटों द्वारा चुना जाता है, और $ K_t = \ bar {k} _t $ (सभी $ k_t $ का औसत)।

अब, एजेंट सक्रिय रूप से अधिकतम उपयोगिता (कुछ बाधाओं के तहत) को अधिकतम करना चाहते हैं और उनके पास सीआरआरए (निरंतर रिश्तेदार जोखिम से बचने) उपयोगिता है, इसलिए अधिकतमकरण जैसा दिखता है: $$ \ sum_ {t = 0} ^ \ infty \ बीटा ^ t \ बिग (\ frac {c_t ^ {1- \ गामा}} {1- \ गामा} \ बिग) $$ $$ s.t \;। Y_t = k_t ^ \ अल्फा (E_tL) ^ {1- \ अल्फा} $$ $$ c_t + i_t = Y_t $$ $$ k_ {t + 1} = (1- \ डेल्टा) k_t + i_t $$ $$ c_t, i_t \ geq0 $$

$ E_tL $ प्रभावी श्रम है और बाकी चर विशिष्ट हैं (यदि अनुरोध किया गया है तो मैं उनकी परिभाषा दे सकता हूं)।

मॉडल के लिए एक अंतिम जोड़ यह है कि दो संतुलन बाधाएं हैं: $$ E_t = \ frac {K_t} {एल} $$ $$ k_t = K_t $$ $ \ Textbf {समाधान:} $

एक यूलर समीकरण दृष्टिकोण का उपयोग करना, उद्देश्य में दो शब्द: $ $ ... \ frac {\ बीटा ^ t [k_t ^ \ अल्फा K_t ^ {1- \ अल्फा} + (1- \ डेल्टा) k_t-k_ {t + 1}] ^ {1- \ "गामा}} { १ / १ गामा} + \ frac {\ बीटा ^ {t + १} [k_ {t + १} ^ \ अल्फ़ा K_ {t + १} ^ {१/१ अल्फ़ा} + (1- \ डेल्टा) k_ {t +1} -k_ {टी + 2}] ^ {1- \ गामा}} {1- \ गामा} ... $$ एफओसी:

w.r.t $ k_ {t + 1} $ और खपत में प्रतिस्थापन:

$ $ \ बीटा ^ tc_t ^ {- \ Gamma} = \ beta ^ {t + 1} c_ {t + 1} ^ {- \ Gamma} [\ अल्फा k_ {t + 1} ^ {\ alpha-1 }___ {t + 1} ^ {1- \ अल्फा} + 1- \ डेल्टा] $$ संतुलन बाधाओं में स्थानापन्न: $ $ c_t ^ {- \ Gamma} = \ beta c_ {t + 1} ^ {- \ Gamma} [\ अल्फा + 1- \ डेल्टा] $ $ $ $ \ _ का अर्थ है \ frac {c_ {t + 1}} {c_t} = [\ Beta (\ Alpha + 1- \ delta)] ^ {\ frac {1} {\ gamma}} $ $ इसका मतलब यह है कि खपत निरंतर दर से बढ़ती है जो वरीयता मापदंडों पर निर्भर करती है।

अगली बात मैं यह साबित करना चाहता हूं कि हमारे पास एक संतुलित विकास पथ है। इसके द्वारा मेरा मतलब है कि सभी चर एक ही स्थिर दर से बढ़ते हैं। $$ \ frac {{k_ t + 1}} {} k_t = \ frac {{c_ t + 1}} {} c_t? $$ मैंने सवाल का जवाब देना शुरू कर दिया है, लेकिन मैं अटक गया हूं। यह है, जो कि अभी तक मेरे पास है: $$ k_ {t + 1} = k_t ^ \ अल्फा K_t ^ {1- \ Alpha} + (1- \ डेल्टा) k_t-c_t $$ संतुलन में $ K_t = k_t $: $$ k_ {t + 1} = k_t + (1- \ डेल्टा) k_t-c_t $$ $ $ \ _ का अर्थ है \ frac {k_ {t + 1}} {k_t} = 1 + (1- \ डेल्टा) - \ frac {c_t} {k_t} $ $ यदि हमारे पास पूंजी वृद्धि की निरंतर दर है, तो मान लें: $$ \ frac {{k_ t + 1}} {} k_t & LT; \ frac {{c_ t + 1}} {} c_t $$ अगर यह सच है: $$ \ underset {t \ rightarrow \ infty} {lim} \; \ frac {k_ {t + 1}} {k_t} = \ underset {t \ rightarrow \ infty} {lim} \; 1+ (1- \ _) डेल्टा) - \ frac {c_t} {k_t} = - \ infty $$ इसका मतलब है कि विकास दर घटती-बढ़ती रहेगी। अब दो तरीके हो सकते हैं। पहला तरीका है यदि $ \ underset {t \ rightarrow \ infty} {lim} \; k_t = - \ infty $, जो स्पष्ट रूप से दिखाएगा कि यह मामला नहीं हो सकता क्योंकि पूंजी नकारात्मक नहीं हो सकती। दूसरा तरीका जो यह हो सकता है यदि $ \ underset {t \ rightarrow \ infty} {lim} \; k_t = D $ जहां $ D $ कुछ सकारात्मक क्षैतिज असममित है। यदि इसने कुछ विषमता का रुख किया, तो कोई कारण नहीं है कि ऐसा नहीं हो सकता है कि खपत पूंजी की तुलना में तेज दर से बढ़ती है। यह वह जगह है जहां मैं फंस गया हूं। मैं यह कैसे दिखा सकता हूं कि यह कुछ asymptote में परिवर्तित नहीं हो सकता है, और यह $ \ अंडरस्सेट {t \ rightarrow \ infty} {lim} \; k_t = - \ infty $ है? इसके अलावा, अगर यह दिखाने का एक आसान तरीका है कि यह मॉडल संतुलित विकास पथ प्रदर्शित करता है, तो यह क्या है? किसी भी तरह की सहायता का स्वागत किया जाएगा!

जवाबों:


3

आपने प्राप्त कर लिया है

$$ \ frac {c_ {t + 1}} {c_t} = [\ Beta (\ Alpha + 1- \ delta)] ^ {\ frac {1} {\ gamma}} \ equiv 1 + g $ $

तथा

$$ \ frac {{k_ t + 1}} {} k_t = 1 + (1- \ डेल्टा) - \ frac {c_t} {} k_t $$

बराबरी करके आप दिखा सकते हैं कि एक अनूठा नियम है जो संतुलित विकास पथ को बनाए रखता है

$$ \ frac {k_ {t + 1}} {k_t} = \ frac {c_ {t + 1}} {c_t} \ implies c_t = (1- \ delta-g) k_t $$

(बहुत अधिक खपत, वैसे)। इससे पता चलता है कि मॉडल में एक अद्वितीय संतुलित विकास पथ है।

यदि आप आगे बहस करना चाहते हैं कि अर्थव्यवस्था वास्तव में इस रास्ते को चुनेगी, तो आपको ट्रांसवर्सिटी कंडीशन (जो कि एक चुने हुए मार्ग को पूंजी संचय पर होनी चाहिए) के लिए बाध्य करती है, और शायद इनडा शर्त है कि आपकी चुनी हुई उपयोगिता संक्षिप्त हो।


क्या $ g $ केवल एक स्थिर है या इसका कोई विशिष्ट अर्थ है?
DornerA

@ डोनर यूलर समीकरण के माध्यम से प्राप्त उपभोग की निरंतर वृद्धि दर को लिखने के लिए सिर्फ एक कॉम्पैक्ट तरीका है।
Alecos Papadopoulos
हमारी साइट का प्रयोग करके, आप स्वीकार करते हैं कि आपने हमारी Cookie Policy और निजता नीति को पढ़ और समझा लिया है।
Licensed under cc by-sa 3.0 with attribution required.